[Toán 8] Bất đẳng thức khó.

C

congtu_ho_nguyen

ap dung BDT /sqrt{a+b}<= /sqrt{a}+/sqrt{b}(CM bai 28 trong SGK lop 9) ta co

a+b−c√<=a√+b√−c√ \Rightarrow a+b−c√/a√+b√−c√<=1
TT
b+c−a√<=b√+c√−a√ \Rightarrow b+c−a√/b√+c√−a√ <=1
c+a−b√ <=c√+a√−b√ \Rightarrow c+a−b√ /c√+a√−b√<=1
cong tung ve BDT vao ta duoc đpcm.
 
H

huynhbachkhoa23

ap dung BDT /sqrt{a+b}<= /sqrt{a}+/sqrt{b}(CM bai 28 trong SGK lop 9) ta co

a+b−c√<=a√+b√−c√ \Rightarrow a+b−c√/a√+b√−c√<=1
TT
b+c−a√<=b√+c√−a√ \Rightarrow b+c−a√/b√+c√−a√ <=1
c+a−b√ <=c√+a√−b√ \Rightarrow c+a−b√ /c√+a√−b√<=1
cong tung ve BDT vao ta duoc đpcm.

BDT $\sqrt{a+b} \ge \sqrt{a}+\sqrt{b}$ có điểm rơi tại $ab=0$

Vậy tam giác đó có cạnh bằng $0$ à :)|
 
H

haiyen621

Đội 1
Vì a, b, c là độ dài 3 cạnh tam giác \Rightarrow a, b, c > 0

Ta có công thức [TEX]\sqrt[]{a}-\sqrt[]{b} \leq \sqrt[]{a-b}[/TEX] ( Bình phương 2 vế để CM )

[TEX]\Rightarrow \sqrt[]{a+b-c} \geq \sqrt[]{a+b}-\sqrt[]{c}[/TEX]

[TEX]\Rightarrow \frac{\sqrt{a+b-c}}{\sqrt{a}+\sqrt{b}-\sqrt{c}} \leq 1[/TEX]

Tương tự. ...

Sau đó cộng từng vế vào được đpcm

@transformer123: sai rồi
 
Last edited by a moderator:
R

riverflowsinyou1

Lời giải của 1 bác ở Colombia :) mọi người tham khảo mình thì mình không đọc đâu =))
Tenemos que $\sqrt a+\sqrt b > \sqrt{a+b}\ge \sqrt c$ y lo mismo para las ciclicas. Esto quiere decir que $\sqrt a , \sqrt b , \sqrt c$ son los lados de un triangulo también. Entonces podemos hacer la sustitución de Ravi es decir que podemos poner:
$\sqrt a =x+y$
$\sqrt b =y+z$
$\sqrt c =z+x$

Con $x,y,z$ reales positivos. Por lo tanto la desigualdad es equivalente a:
$\sum_{cyc}\frac{\sqrt{(x+y)^{2}+(y+z)^{2}-(z+x)^{2}}}{2y}\le 3$
Abriendo los cuadrados y multiplicando por $2xyz$ a ambos lados obtenemos que la desigualdad es igual a:

$\sum_{cyc}\sqrt{2x^{2}yz^{2}(x+y+z)-2x^{3}z^{3}}\le 6xyz$

Usando Jensen (con $f(a)= \sqrt{a}$ que es concava) obtenemos que es suficiente probar que:

$\sum_{cyc}\sqrt{2x^{2}yz^{2}(x+y+z)-2x^{3}z^{3}}\le 3\sqrt{\frac{2(x+y+z)(\sum_{cyc}x^{2}y^{2}z)-2(\sum_{cyc}x^{3}y^{3})}{3}}\le 6xyz$

Esto ultimo se tiene si y solo si:
$\frac{(x+y+z)(\sum_{cyc}x^{2}y^{2}z)-(\sum_{cyc}x^{3}y^{3})}{3}\le 2x^{2}y^{2}z^{2}$

Que es equivalente (después de expandir y poner todo del mismo lado) a:

$\frac{1}{2}\sum_{sym}x^{2}y^{2}z^{2}+\frac{1}{2}\sum_{sym}x^{3}y^{3}-\sum_{sym}x^{3}y^{2}z = \sum_{cyc}xy(xy-yz)(xy-zx) \ge 0$

Que no es otra cosa que la conocida desigualdad de Schur.
 
Top Bottom